Probability of an event for a continuous random vector of three coordinates












0












$begingroup$


Let $X=(X_1,X_2,X_3)$ be a continuous random vector of the joint pdf $$f(x_1,x_2,x_3)= 12x_2 ;mathrm f mathrm o mathrm r ; 0<x_3<x_2<x_1<1$$ and $0$ elsewhere.



I need to find the probability of event $B$ where $B={x_3leq1/3}$. I've been able to sketch the support and see that it's a sort of pyramidal shape, and know that I should be able to find the probability via a triple integral.



My instinct says to simply set it up like so:
$$int_0^{1/3}int_{x_3}^{x_1}int_{x_2}^{1}12x_2;dx_1dx_2dx_3$$



but this is clearly wrong since the answer will still be in terms of a variable. If anyone could help me understand where I've been going wrong it would be greatly appreciated!










share|cite|improve this question











$endgroup$

















    0












    $begingroup$


    Let $X=(X_1,X_2,X_3)$ be a continuous random vector of the joint pdf $$f(x_1,x_2,x_3)= 12x_2 ;mathrm f mathrm o mathrm r ; 0<x_3<x_2<x_1<1$$ and $0$ elsewhere.



    I need to find the probability of event $B$ where $B={x_3leq1/3}$. I've been able to sketch the support and see that it's a sort of pyramidal shape, and know that I should be able to find the probability via a triple integral.



    My instinct says to simply set it up like so:
    $$int_0^{1/3}int_{x_3}^{x_1}int_{x_2}^{1}12x_2;dx_1dx_2dx_3$$



    but this is clearly wrong since the answer will still be in terms of a variable. If anyone could help me understand where I've been going wrong it would be greatly appreciated!










    share|cite|improve this question











    $endgroup$















      0












      0








      0





      $begingroup$


      Let $X=(X_1,X_2,X_3)$ be a continuous random vector of the joint pdf $$f(x_1,x_2,x_3)= 12x_2 ;mathrm f mathrm o mathrm r ; 0<x_3<x_2<x_1<1$$ and $0$ elsewhere.



      I need to find the probability of event $B$ where $B={x_3leq1/3}$. I've been able to sketch the support and see that it's a sort of pyramidal shape, and know that I should be able to find the probability via a triple integral.



      My instinct says to simply set it up like so:
      $$int_0^{1/3}int_{x_3}^{x_1}int_{x_2}^{1}12x_2;dx_1dx_2dx_3$$



      but this is clearly wrong since the answer will still be in terms of a variable. If anyone could help me understand where I've been going wrong it would be greatly appreciated!










      share|cite|improve this question











      $endgroup$




      Let $X=(X_1,X_2,X_3)$ be a continuous random vector of the joint pdf $$f(x_1,x_2,x_3)= 12x_2 ;mathrm f mathrm o mathrm r ; 0<x_3<x_2<x_1<1$$ and $0$ elsewhere.



      I need to find the probability of event $B$ where $B={x_3leq1/3}$. I've been able to sketch the support and see that it's a sort of pyramidal shape, and know that I should be able to find the probability via a triple integral.



      My instinct says to simply set it up like so:
      $$int_0^{1/3}int_{x_3}^{x_1}int_{x_2}^{1}12x_2;dx_1dx_2dx_3$$



      but this is clearly wrong since the answer will still be in terms of a variable. If anyone could help me understand where I've been going wrong it would be greatly appreciated!







      probability probability-distributions






      share|cite|improve this question















      share|cite|improve this question













      share|cite|improve this question




      share|cite|improve this question








      edited Jan 20 at 21:03







      sk13

















      asked Jan 20 at 20:25









      sk13sk13

      246




      246






















          2 Answers
          2






          active

          oldest

          votes


















          1












          $begingroup$

          It is $int_0^{1/3} int_{x_3}^{1} int_{x_2}^{1} 12x_2 dx_1dx_2dx_3$. (The middle integral cannot involve $x_1$. The condition $x_2 <x_1$ is already taken care of in the limits for $x_1$).






          share|cite|improve this answer









          $endgroup$





















            0












            $begingroup$

            Change the last integral from ($x_2$ to 1) to ($x_1$ to 1). Does it make any sense?






            share|cite|improve this answer









            $endgroup$













            • $begingroup$
              Sorry, I'm not sure I understand. Why would the bounds of $x_1$ be ($x_1$,1)?
              $endgroup$
              – sk13
              Jan 20 at 22:49













            Your Answer





            StackExchange.ifUsing("editor", function () {
            return StackExchange.using("mathjaxEditing", function () {
            StackExchange.MarkdownEditor.creationCallbacks.add(function (editor, postfix) {
            StackExchange.mathjaxEditing.prepareWmdForMathJax(editor, postfix, [["$", "$"], ["\\(","\\)"]]);
            });
            });
            }, "mathjax-editing");

            StackExchange.ready(function() {
            var channelOptions = {
            tags: "".split(" "),
            id: "69"
            };
            initTagRenderer("".split(" "), "".split(" "), channelOptions);

            StackExchange.using("externalEditor", function() {
            // Have to fire editor after snippets, if snippets enabled
            if (StackExchange.settings.snippets.snippetsEnabled) {
            StackExchange.using("snippets", function() {
            createEditor();
            });
            }
            else {
            createEditor();
            }
            });

            function createEditor() {
            StackExchange.prepareEditor({
            heartbeatType: 'answer',
            autoActivateHeartbeat: false,
            convertImagesToLinks: true,
            noModals: true,
            showLowRepImageUploadWarning: true,
            reputationToPostImages: 10,
            bindNavPrevention: true,
            postfix: "",
            imageUploader: {
            brandingHtml: "Powered by u003ca class="icon-imgur-white" href="https://imgur.com/"u003eu003c/au003e",
            contentPolicyHtml: "User contributions licensed under u003ca href="https://creativecommons.org/licenses/by-sa/3.0/"u003ecc by-sa 3.0 with attribution requiredu003c/au003e u003ca href="https://stackoverflow.com/legal/content-policy"u003e(content policy)u003c/au003e",
            allowUrls: true
            },
            noCode: true, onDemand: true,
            discardSelector: ".discard-answer"
            ,immediatelyShowMarkdownHelp:true
            });


            }
            });














            draft saved

            draft discarded


















            StackExchange.ready(
            function () {
            StackExchange.openid.initPostLogin('.new-post-login', 'https%3a%2f%2fmath.stackexchange.com%2fquestions%2f3081095%2fprobability-of-an-event-for-a-continuous-random-vector-of-three-coordinates%23new-answer', 'question_page');
            }
            );

            Post as a guest















            Required, but never shown

























            2 Answers
            2






            active

            oldest

            votes








            2 Answers
            2






            active

            oldest

            votes









            active

            oldest

            votes






            active

            oldest

            votes









            1












            $begingroup$

            It is $int_0^{1/3} int_{x_3}^{1} int_{x_2}^{1} 12x_2 dx_1dx_2dx_3$. (The middle integral cannot involve $x_1$. The condition $x_2 <x_1$ is already taken care of in the limits for $x_1$).






            share|cite|improve this answer









            $endgroup$


















              1












              $begingroup$

              It is $int_0^{1/3} int_{x_3}^{1} int_{x_2}^{1} 12x_2 dx_1dx_2dx_3$. (The middle integral cannot involve $x_1$. The condition $x_2 <x_1$ is already taken care of in the limits for $x_1$).






              share|cite|improve this answer









              $endgroup$
















                1












                1








                1





                $begingroup$

                It is $int_0^{1/3} int_{x_3}^{1} int_{x_2}^{1} 12x_2 dx_1dx_2dx_3$. (The middle integral cannot involve $x_1$. The condition $x_2 <x_1$ is already taken care of in the limits for $x_1$).






                share|cite|improve this answer









                $endgroup$



                It is $int_0^{1/3} int_{x_3}^{1} int_{x_2}^{1} 12x_2 dx_1dx_2dx_3$. (The middle integral cannot involve $x_1$. The condition $x_2 <x_1$ is already taken care of in the limits for $x_1$).







                share|cite|improve this answer












                share|cite|improve this answer



                share|cite|improve this answer










                answered Jan 20 at 23:58









                Kavi Rama MurthyKavi Rama Murthy

                64.7k42766




                64.7k42766























                    0












                    $begingroup$

                    Change the last integral from ($x_2$ to 1) to ($x_1$ to 1). Does it make any sense?






                    share|cite|improve this answer









                    $endgroup$













                    • $begingroup$
                      Sorry, I'm not sure I understand. Why would the bounds of $x_1$ be ($x_1$,1)?
                      $endgroup$
                      – sk13
                      Jan 20 at 22:49


















                    0












                    $begingroup$

                    Change the last integral from ($x_2$ to 1) to ($x_1$ to 1). Does it make any sense?






                    share|cite|improve this answer









                    $endgroup$













                    • $begingroup$
                      Sorry, I'm not sure I understand. Why would the bounds of $x_1$ be ($x_1$,1)?
                      $endgroup$
                      – sk13
                      Jan 20 at 22:49
















                    0












                    0








                    0





                    $begingroup$

                    Change the last integral from ($x_2$ to 1) to ($x_1$ to 1). Does it make any sense?






                    share|cite|improve this answer









                    $endgroup$



                    Change the last integral from ($x_2$ to 1) to ($x_1$ to 1). Does it make any sense?







                    share|cite|improve this answer












                    share|cite|improve this answer



                    share|cite|improve this answer










                    answered Jan 20 at 20:43









                    ShaqShaq

                    3049




                    3049












                    • $begingroup$
                      Sorry, I'm not sure I understand. Why would the bounds of $x_1$ be ($x_1$,1)?
                      $endgroup$
                      – sk13
                      Jan 20 at 22:49




















                    • $begingroup$
                      Sorry, I'm not sure I understand. Why would the bounds of $x_1$ be ($x_1$,1)?
                      $endgroup$
                      – sk13
                      Jan 20 at 22:49


















                    $begingroup$
                    Sorry, I'm not sure I understand. Why would the bounds of $x_1$ be ($x_1$,1)?
                    $endgroup$
                    – sk13
                    Jan 20 at 22:49






                    $begingroup$
                    Sorry, I'm not sure I understand. Why would the bounds of $x_1$ be ($x_1$,1)?
                    $endgroup$
                    – sk13
                    Jan 20 at 22:49




















                    draft saved

                    draft discarded




















































                    Thanks for contributing an answer to Mathematics Stack Exchange!


                    • Please be sure to answer the question. Provide details and share your research!

                    But avoid



                    • Asking for help, clarification, or responding to other answers.

                    • Making statements based on opinion; back them up with references or personal experience.


                    Use MathJax to format equations. MathJax reference.


                    To learn more, see our tips on writing great answers.




                    draft saved


                    draft discarded














                    StackExchange.ready(
                    function () {
                    StackExchange.openid.initPostLogin('.new-post-login', 'https%3a%2f%2fmath.stackexchange.com%2fquestions%2f3081095%2fprobability-of-an-event-for-a-continuous-random-vector-of-three-coordinates%23new-answer', 'question_page');
                    }
                    );

                    Post as a guest















                    Required, but never shown





















































                    Required, but never shown














                    Required, but never shown












                    Required, but never shown







                    Required, but never shown

































                    Required, but never shown














                    Required, but never shown












                    Required, but never shown







                    Required, but never shown







                    Popular posts from this blog

                    Can a sorcerer learn a 5th-level spell early by creating spell slots using the Font of Magic feature?

                    Does disintegrating a polymorphed enemy still kill it after the 2018 errata?

                    A Topological Invariant for $pi_3(U(n))$